LSAT and Law School Admissions Forum

Get expert LSAT preparation and law school admissions advice from PowerScore Test Preparation.

 Administrator
PowerScore Staff
  • PowerScore Staff
  • Posts: 8917
  • Joined: Feb 02, 2011
|
#35363
Complete Question Explanation

Weaken. The correct answer choice is (B)

The company spokesperson argues that the Filterator X water filter is effective because its owners are
satisfied with the product’s performance. This stimulus contains a typical argument/counterargument
structure: it introduces an assertion (the Household Products criticism of Filterator X), which is later
rejected (“this attack … is undermined”) and specific evidence for the objection is provided (“owners
… are satisfied”). Because there are two arguments to keep track of, make sure to identify an answer
choice that undermines the correct argument, i.e. the company spokesperson’s. To weaken it, show
that the water filter is not necessarily effective in spite of its owners’ alleged satisfaction.

The spokesperson’s conclusion is severely flawed. First off, the type of anecdotal evidence she relies
upon is somewhat dubious, because we have no way of knowing how the company measured the
satisfaction of its customers. Also, since the total number of Filterator X owners is unknown, the
percentage of those satisfied with the product’s performance is also unknown: if 20 million people
use Filterator X, the fact that, say, 2 million of them are satisfied means very little. Most importantly,
even if the vast majority of owners are satisfied, that still does not prove that the product is effective
in removing chemical contaminants from the water. It is entirely possible, for instance, that Filterator
X owners cannot tell if their product works as advertised, making their satisfaction a debatable proxy
for quality. Generally speaking, the LSAT tends to avoid weakening arguments by attacking the
factual veracity of its premises. This is why the last line of criticism is likely to provide us with the
strongest possible prephrase.

Answer choice (A): Whether Filterator X improves the taste of drinking water is irrelevant: the issue
is whether the water filter effectively removes chemical contaminants. Furthermore, this answer
choice attacks the Household Products’ position, not the spokesperson’s.

Answer choice (B): This is the correct answer choice. If most Filterator X owners cannot reliably
determine the effectiveness of their filter, it is entirely possible that the filter is not effective in
removing chemical contaminants from water, and that the owners’ satisfaction means very little. This
would strengthen the magazine’s claims and, in turn, weaken the spokesperson’s argument.

Answer choice (C): The fact that households with contaminated water are more likely than others
to buy Filterator X does not weaken, and might even strengthen the spokesperson’s argument. If
these households were especially motivated to find an effective product, this would corroborate the
spokesperson’s claims that Filterator X is a high-quality product.

Answer choice (D): The number of Filterator X owners who read Household Products on a regular
basis is entirely irrelevant to the author’s conclusion.

Answer choice (E): The fact that Household Products’ evaluations of Filterator X have been
consistently negative could be interpreted in several ways: It could mean that the filter is indeed of
poor quality, but it is also possible that the magazine is simply biased in their evaluations. Ultimately,
just because this one magazine has never been a fan of Filterator X means very little. This answer
choice fails to engage the spokesperson’s counterargument and attacks none of its inherent flaws.

Get the most out of your LSAT Prep Plus subscription.

Analyze and track your performance with our Testing and Analytics Package.